Gradient of a function points in the direction of the greatest rate of increase of the function.

The name of the pictureThe name of the pictureThe name of the pictureClash Royale CLAN TAG#URR8PPP











up vote
1
down vote

favorite
1












Somewhere I saw that the gradient of a function points in the direction of the greatest rate of increase of the function, and its magnitude is the slope of the graph in that direction. I don't understand it at all. Can somebody help me to understand it? Please give some example?










share|cite|improve this question





















  • Possible duplicate of math.stackexchange.com/questions/223252/…
    – saulspatz
    Sep 6 at 5:44










  • You might also have a look at math.stackexchange.com/q/1912660/265466.
    – amd
    Sep 6 at 6:39














up vote
1
down vote

favorite
1












Somewhere I saw that the gradient of a function points in the direction of the greatest rate of increase of the function, and its magnitude is the slope of the graph in that direction. I don't understand it at all. Can somebody help me to understand it? Please give some example?










share|cite|improve this question





















  • Possible duplicate of math.stackexchange.com/questions/223252/…
    – saulspatz
    Sep 6 at 5:44










  • You might also have a look at math.stackexchange.com/q/1912660/265466.
    – amd
    Sep 6 at 6:39












up vote
1
down vote

favorite
1









up vote
1
down vote

favorite
1






1





Somewhere I saw that the gradient of a function points in the direction of the greatest rate of increase of the function, and its magnitude is the slope of the graph in that direction. I don't understand it at all. Can somebody help me to understand it? Please give some example?










share|cite|improve this question













Somewhere I saw that the gradient of a function points in the direction of the greatest rate of increase of the function, and its magnitude is the slope of the graph in that direction. I don't understand it at all. Can somebody help me to understand it? Please give some example?







calculus multivariable-calculus






share|cite|improve this question













share|cite|improve this question











share|cite|improve this question




share|cite|improve this question










asked Sep 6 at 5:29









Infinity

518213




518213











  • Possible duplicate of math.stackexchange.com/questions/223252/…
    – saulspatz
    Sep 6 at 5:44










  • You might also have a look at math.stackexchange.com/q/1912660/265466.
    – amd
    Sep 6 at 6:39
















  • Possible duplicate of math.stackexchange.com/questions/223252/…
    – saulspatz
    Sep 6 at 5:44










  • You might also have a look at math.stackexchange.com/q/1912660/265466.
    – amd
    Sep 6 at 6:39















Possible duplicate of math.stackexchange.com/questions/223252/…
– saulspatz
Sep 6 at 5:44




Possible duplicate of math.stackexchange.com/questions/223252/…
– saulspatz
Sep 6 at 5:44












You might also have a look at math.stackexchange.com/q/1912660/265466.
– amd
Sep 6 at 6:39




You might also have a look at math.stackexchange.com/q/1912660/265466.
– amd
Sep 6 at 6:39










1 Answer
1






active

oldest

votes

















up vote
0
down vote













The link I gave has very good answers based on the directional derivative that prove the gradient points in the direction of steepest ascent, using directional derivatives. What has made this result intuitive for me is another result: the gradient is perpendicular to the level curve. Imagine standing on a hillside, and visualize the curve passing through all points at the same altitude as the point you're standing at. It just seems clear to me from experience, that the steepest climb is perpendicular to that curve (the direction of the gradient), and that if if you were to set a ball on the ground and let it go, it would roll downhill perpendicular to that curve (in the direction of the negative of the gradient.)



The link I gave proves the theorem for a function of $3$ variables, but the same proof works in any number of dimensions of course.






share|cite|improve this answer




















    Your Answer




    StackExchange.ifUsing("editor", function ()
    return StackExchange.using("mathjaxEditing", function ()
    StackExchange.MarkdownEditor.creationCallbacks.add(function (editor, postfix)
    StackExchange.mathjaxEditing.prepareWmdForMathJax(editor, postfix, [["$", "$"], ["\\(","\\)"]]);
    );
    );
    , "mathjax-editing");

    StackExchange.ready(function()
    var channelOptions =
    tags: "".split(" "),
    id: "69"
    ;
    initTagRenderer("".split(" "), "".split(" "), channelOptions);

    StackExchange.using("externalEditor", function()
    // Have to fire editor after snippets, if snippets enabled
    if (StackExchange.settings.snippets.snippetsEnabled)
    StackExchange.using("snippets", function()
    createEditor();
    );

    else
    createEditor();

    );

    function createEditor()
    StackExchange.prepareEditor(
    heartbeatType: 'answer',
    convertImagesToLinks: true,
    noModals: false,
    showLowRepImageUploadWarning: true,
    reputationToPostImages: 10,
    bindNavPrevention: true,
    postfix: "",
    noCode: true, onDemand: true,
    discardSelector: ".discard-answer"
    ,immediatelyShowMarkdownHelp:true
    );



    );













     

    draft saved


    draft discarded


















    StackExchange.ready(
    function ()
    StackExchange.openid.initPostLogin('.new-post-login', 'https%3a%2f%2fmath.stackexchange.com%2fquestions%2f2907123%2fgradient-of-a-function-points-in-the-direction-of-the-greatest-rate-of-increase%23new-answer', 'question_page');

    );

    Post as a guest






























    1 Answer
    1






    active

    oldest

    votes








    1 Answer
    1






    active

    oldest

    votes









    active

    oldest

    votes






    active

    oldest

    votes








    up vote
    0
    down vote













    The link I gave has very good answers based on the directional derivative that prove the gradient points in the direction of steepest ascent, using directional derivatives. What has made this result intuitive for me is another result: the gradient is perpendicular to the level curve. Imagine standing on a hillside, and visualize the curve passing through all points at the same altitude as the point you're standing at. It just seems clear to me from experience, that the steepest climb is perpendicular to that curve (the direction of the gradient), and that if if you were to set a ball on the ground and let it go, it would roll downhill perpendicular to that curve (in the direction of the negative of the gradient.)



    The link I gave proves the theorem for a function of $3$ variables, but the same proof works in any number of dimensions of course.






    share|cite|improve this answer
























      up vote
      0
      down vote













      The link I gave has very good answers based on the directional derivative that prove the gradient points in the direction of steepest ascent, using directional derivatives. What has made this result intuitive for me is another result: the gradient is perpendicular to the level curve. Imagine standing on a hillside, and visualize the curve passing through all points at the same altitude as the point you're standing at. It just seems clear to me from experience, that the steepest climb is perpendicular to that curve (the direction of the gradient), and that if if you were to set a ball on the ground and let it go, it would roll downhill perpendicular to that curve (in the direction of the negative of the gradient.)



      The link I gave proves the theorem for a function of $3$ variables, but the same proof works in any number of dimensions of course.






      share|cite|improve this answer






















        up vote
        0
        down vote










        up vote
        0
        down vote









        The link I gave has very good answers based on the directional derivative that prove the gradient points in the direction of steepest ascent, using directional derivatives. What has made this result intuitive for me is another result: the gradient is perpendicular to the level curve. Imagine standing on a hillside, and visualize the curve passing through all points at the same altitude as the point you're standing at. It just seems clear to me from experience, that the steepest climb is perpendicular to that curve (the direction of the gradient), and that if if you were to set a ball on the ground and let it go, it would roll downhill perpendicular to that curve (in the direction of the negative of the gradient.)



        The link I gave proves the theorem for a function of $3$ variables, but the same proof works in any number of dimensions of course.






        share|cite|improve this answer












        The link I gave has very good answers based on the directional derivative that prove the gradient points in the direction of steepest ascent, using directional derivatives. What has made this result intuitive for me is another result: the gradient is perpendicular to the level curve. Imagine standing on a hillside, and visualize the curve passing through all points at the same altitude as the point you're standing at. It just seems clear to me from experience, that the steepest climb is perpendicular to that curve (the direction of the gradient), and that if if you were to set a ball on the ground and let it go, it would roll downhill perpendicular to that curve (in the direction of the negative of the gradient.)



        The link I gave proves the theorem for a function of $3$ variables, but the same proof works in any number of dimensions of course.







        share|cite|improve this answer












        share|cite|improve this answer



        share|cite|improve this answer










        answered Sep 6 at 6:07









        saulspatz

        11.6k21324




        11.6k21324



























             

            draft saved


            draft discarded















































             


            draft saved


            draft discarded














            StackExchange.ready(
            function ()
            StackExchange.openid.initPostLogin('.new-post-login', 'https%3a%2f%2fmath.stackexchange.com%2fquestions%2f2907123%2fgradient-of-a-function-points-in-the-direction-of-the-greatest-rate-of-increase%23new-answer', 'question_page');

            );

            Post as a guest













































































            這個網誌中的熱門文章

            Is there any way to eliminate the singular point to solve this integral by hand or by approximations?

            Why am i infinitely getting the same tweet with the Twitter Search API?

            Carbon dioxide